Cesaro summability implies bounded partial sums

Click For Summary
If the series ∑c_n is Cesaro summable and c_n > 0 for all n ≥ 0, then the partial sums S_N must be bounded. The discussion emphasizes the need to establish an upper bound for these partial sums, which are nondecreasing due to the positivity of c_n. The existence of a least upper bound is crucial, as it relates to the limit of the partial sums. Ultimately, this limit corresponds to the traditional sum of the series. The conclusion is that boundedness of partial sums follows from Cesaro summability in this context.
stripes
Messages
262
Reaction score
0

Homework Statement



Suppose c_{n} > 0 for each n\geq 0. Prove that if \sum ^{\infty}_{n=0} c_{n} is Cesaro summable, then the partial sums S_{N} are bounded.

Homework Equations



--

The Attempt at a Solution



I tried contraposition; that was getting me nowhere. I have a few inequalities here and there but they don't tell me anything. I need to show that there exists an upperbound for the partial sums. This means there exists a least upperbound. I need to find that least upperbound. Becausec_{n} > 0 for each n\geq 0, then the series is nondecreasing, which means the partial sums are nondecreasing, so we are looking for an upperbound, not a lowerbound.
 
Physics news on Phys.org
stripes said:

Homework Statement



Suppose c_{n} > 0 for each n\geq 0. Prove that if \sum ^{\infty}_{n=0} c_{n} is Cesaro summable, then the partial sums S_{N} are bounded.

Homework Equations



--

The Attempt at a Solution



I tried contraposition; that was getting me nowhere. I have a few inequalities here and there but they don't tell me anything. I need to show that there exists an upperbound for the partial sums. This means there exists a least upperbound. I need to find that least upperbound.

That least upper bound, if it exists, is \lim_{n \to \infty} S_n, which is the definition of \sum_{n=0}^{\infty} c_n in the traditional sense.
 
Question: A clock's minute hand has length 4 and its hour hand has length 3. What is the distance between the tips at the moment when it is increasing most rapidly?(Putnam Exam Question) Answer: Making assumption that both the hands moves at constant angular velocities, the answer is ## \sqrt{7} .## But don't you think this assumption is somewhat doubtful and wrong?

Similar threads

  • · Replies 1 ·
Replies
1
Views
2K
  • · Replies 7 ·
Replies
7
Views
2K
  • · Replies 1 ·
Replies
1
Views
5K
  • · Replies 2 ·
Replies
2
Views
2K
  • · Replies 12 ·
Replies
12
Views
3K
  • · Replies 3 ·
Replies
3
Views
1K
Replies
2
Views
5K
  • · Replies 7 ·
Replies
7
Views
2K
  • · Replies 3 ·
Replies
3
Views
2K
Replies
1
Views
2K